LSAT and Law School Admissions Forum

Get expert LSAT preparation and law school admissions advice from PowerScore Test Preparation.

 Administrator
PowerScore Staff
  • PowerScore Staff
  • Posts: 8929
  • Joined: Feb 02, 2011
|
#59027
Please post your questions below!
 afulbright.2000@gmail.com
  • Posts: 13
  • Joined: Apr 28, 2021
|
#96900
Administrator wrote: Tue Oct 02, 2018 12:59 pm Please post your questions below!
Hi, can someone explain why B is incorrect?

I can understand why A is correct, but I am still slightly hesitant as to understanding why B is the wrong answer.

Thank you!
User avatar
 atierney
PowerScore Staff
  • PowerScore Staff
  • Posts: 215
  • Joined: Jul 06, 2021
|
#96935
B is actually correct! But let's look at A: The Park Street Cafe closed this year even though its customer base was satisfied. So, because its customer base was satisfied, the only conclusion one can draw is that the Park Street Cafe closed because it was facing strong competition.

Here, there is no necessary condition from which the conclusion is drawn. The stimulus is based on a conditional statement: Munroe's election is necessitated by "a fundamental shift in the sentiments of the electorate and a well-run campaign." With A however, there is no conditional statement at all!

Let me know if you have further questions on this.
 afulbright.2000@gmail.com
  • Posts: 13
  • Joined: Apr 28, 2021
|
#96999
Oh my goodness! My apologies. I meant to ask the opposite of what I had typed...

So, your explanation is definitely helpful. But I still am struggling in understanding the question though.

Maybe it would be helpful for me to share the diagramming I did for the question?

I had initially "graphed" out the sufficient and necessary conditions in the stimulus.
IF Munroe is elected -------> THEN a fundamental shift in the sentiments of the electorate *and* a well-run campaign

For answer (A), I had graphed out the following as well:
IF the Park Street Cafe closed -------> THEN it was facing strong competition

Was I wrong to enforce a sufficient and necessary on answer choice (A)?
Also, could another reason why answer (A) is wrong be because the necessary involves only one condition (strong competition) instead of two like the stimulus doe (shift in sentiments & well-run campaign)?

Thanks in advance for the help!
 Rachael Wilkenfeld
PowerScore Staff
  • PowerScore Staff
  • Posts: 1392
  • Joined: Dec 15, 2011
|
#97059
Exactly, afulbright. In this stimulus we have a sufficient condition that has two necessary conditions. The sufficient condition is met, so we can conclude that one of the necessary conditions is also met.

Answer choice (A) has a few issues. First, there's only one quasi-necessary condition. But also answer choice (A) includes some causal language that is not present in the stimulus. We don't want to add additional reasoning to the argument in a parallel reasoning question. The reasoning in answer choice (A) is causal, not conditional. We'd eliminate it for that reason.

Hope that helps!
User avatar
 akreimerman1
  • Posts: 14
  • Joined: Jul 16, 2023
|
#102605
Hi,

Can someone explain why E is incorrect and why B is correct? I thought E properly explained the two necessary conditions for the sufficient to happen, which would be to not stay open.

Thanks.
User avatar
 Jeff Wren
PowerScore Staff
  • PowerScore Staff
  • Posts: 451
  • Joined: Oct 19, 2022
|
#102957
Hi akreimerman,

Because the argument in the stimulus uses conditional reasoning, it is important to pay careful attention to exactly how the logic flows and keep track of the exact number of (sufficient and necessary) terms.

The best way to do this is to diagram the conditional reasoning in the stimulus, and then find the answer that has an identical diagram/pattern (only with different terms).

The first premise is:

ME

(for Monroe elected)

The second premise is conditional and is diagrammed

ME -> FSS + WRC

(for if Monroe is elected, then there was a fundamental shift in sentiment of the electorate and a well-run campaign)

Note that the conditional indicator of this sentence is "without," which we diagramm using the Unless Equation.

The conclusion is:

FSS

(for there was a fundamental shift in sentiment of the electorate)

We want an answer that follows this exact same pattern/diagram. In other words, a sufficient with two necessary conditions. The sufficient occurs (i.e. gets triggered) in a premise, and then the conclusion is that one of the necessary conditions occurred.

Answer B matches the stimulus perfectly.

CC

(for Cafe closed)

CC -> SC + CBU

(for if Cafe closed, then it faced strong competition and its customer base was unsatisfied)

Note that the conditional indicator of this sentence is "unless," which we diagram using the Unless Equation.

The conclusion is:

SC

(for it faced strong competition)

This diagram is identical in structure to the stimulus.

Answer E has a different structure.

It can be diagrammed:

SO -> LC + SCB

(for if the Cafe stayed open, then there was a lack of competition and a satisfied customer base)

(the contrapositive would be diagrammed)

not LC or not SCB -> not SO

(for if there's no lack of competition or if there's not a satisfied customer based, then the Cafe won't stay open)

another premise states "it had neither," so

not LC and not SCB

and then concludes

not SO

(for not stay open)

As you can see, this diagram is not parallel to the original at all. This argument uses the contrapositive to conclude that the original sufficient didn't occur.

Get the most out of your LSAT Prep Plus subscription.

Analyze and track your performance with our Testing and Analytics Package.